Skip to main content
Log in

Data-Driven Distributionally Robust Risk-Averse Two-Stage Stochastic Linear Programming over Wasserstein Ball

  • Published:
Journal of Optimization Theory and Applications Aims and scope Submit manuscript

Abstract

In this paper, we consider a data-driven distributionally robust two-stage stochastic linear optimization problem over 1-Wasserstein ball centered at a discrete empirical distribution. Differently from the traditional two-stage stochastic programming which involves the expected recourse function as the preference criterion and hence is risk-neutral, we take the conditional value-at-risk (CVaR) as the risk measure in order to model its effects on decision making problems. We mainly explore tractable reformulations for the proposed robust two-stage stochastic programming with mean-CVaR criterion by analyzing the first case where uncertainties are only in the objective function and then the second case where uncertainties are only in the constraints. We demonstrate that the first model can be exactly reformulated as a deterministic convex programming. Furthermore, it is shown that under several different support sets, the resulting convex optimization problems can be converted into computationally tractable conic programmings. Besides, the second model is generally NP-hard since checking constraint feasibility can be reduced to a norm maximization problem over a polytope. However, even with the case of uncertainty in constraints, tractable conic reformulations can be established when the extreme points of the polytope are known. Finally, we present numerical results to discuss how to control the risk for the best decisions and illustrate the computational effectiveness and superiority of the proposed models.

This is a preview of subscription content, log in via an institution to check access.

Access this article

Price excludes VAT (USA)
Tax calculation will be finalised during checkout.

Instant access to the full article PDF.

Fig. 1
Fig. 2

Similar content being viewed by others

Data Availability

Some or all data, models, or code generated or used during the study are available from the second author of the paper by request.

References

  1. Ang, J., Meng, F., Sun, J.: Two-stage stochastic linear programs with incomplete information on uncertainty. Eur. J. Oper. Res. 233(1), 16–22 (2014)

    Article  MathSciNet  Google Scholar 

  2. Artzner, P., Delbaen, F., Eber, J.M., Heath, D.: Coherent measures of risk. Math. Financ. 9(3), 203–228 (1999)

    Article  MathSciNet  Google Scholar 

  3. Ben-Tal, A., Nemirovski, A.: Robust optimization-methodology and applications. Math. Program. 92(3), 453–480 (2002)

    Article  MathSciNet  Google Scholar 

  4. Ben-Tal, A., Teboulle, M.: Expected utility, penalty functions, and duality in stochastic nonlinear programming. Manage. Sci. 32(11), 1445–1466 (1986)

    Article  MathSciNet  Google Scholar 

  5. Bertsimas, D., Doan, X.V., Natarajan, K., Teo, C.P.: Models for minimax stochastic linear optimization problems with risk aversion. Math. Oper. Res. 35(3), 580–602 (2010)

    Article  MathSciNet  Google Scholar 

  6. Bertsimas, D., Shtern, S., Sturt, B.: A data-driven approach to multistage stochastic linear optimization. Manage. Sci. 69(1), 51–74 (2022)

    Article  Google Scholar 

  7. Birge, J.R., Louveaux, F.: Introduction to Stochastic Programming. In: Mikosch, T.V., Resnick, S.I., Robinson, S.M. (eds.) Springer Series in Operations Research and Financial Engineering, 2nd edn. Springer-Verlag, New York (1997)

    Google Scholar 

  8. Blanchet, J., Kang, Y., Murthy, K.: Robust Wasserstein profile inference and applications to machine learning. J. Appl. Probab. 56(3), 830–857 (2019)

    Article  MathSciNet  Google Scholar 

  9. Blanchet, J., Murthy, K.: Quantifying distributional model risk via optimal transport. Math. Oper. Res. 44(2), 565–600 (2019)

    Article  MathSciNet  Google Scholar 

  10. Chen, W., Sim, M., Sun, J., Teo, C.P.: From CVaR to uncertainty set: implications in joint chance-constrained optimization. Oper. Res. 58(2), 470–485 (2009)

    Article  MathSciNet  Google Scholar 

  11. Chen, X., Sim, M., Sun, P., Zhang, J.: A linear decision-based approximation approach to stochastic programming. Oper. Res. 56(2), 344–357 (2007)

    Article  MathSciNet  Google Scholar 

  12. Chen, Z., Xie, W.: Sharing the value-at-risk under distributional ambiguity. Math. Financ. 31(1), 531–559 (2021)

    Article  MathSciNet  Google Scholar 

  13. Delage, E., Ye, Y.: Distributionally robust optimization under moment uncertainty with application to data-driven problems. Oper. Res. 58(3), 595–612 (2010)

    Article  MathSciNet  Google Scholar 

  14. Esfahani, P.M., Kuhn, D.: Data-driven distributionally robust optimization using the Wasserstein metric: performance guarantees and tractable reformulations. Math. Program. 171(1–2), 115–166 (2018)

    Article  MathSciNet  Google Scholar 

  15. Gao, R., Chen, X., Kleywegt, A.J.: Wasserstein distributionally robust optimization and variation regularization. Oper. Res. (2022). https://doi.org/10.1287/opre.2022.2383

    Article  Google Scholar 

  16. Gao, R., Kleywegt, A.J.: Distributionally robust stochastic optimization with Wasserstein distance. Math. Oper. Res. 48(2), 603–655 (2022)

    Article  MathSciNet  Google Scholar 

  17. Gilboa, I., Schmeidler, D.: Maxmin expected utility with non-unique prior. J. Math. Econ. 18(2), 141–153 (1989)

    Article  Google Scholar 

  18. Goh, J., Sim, M.: Distributionally robust optimization and its tractable approximations. Oper. Res. 58(4), 902–917 (2010)

    Article  MathSciNet  Google Scholar 

  19. Hanasusanto, G.A., Kuhn, D.: Conic programming reformulations of two-stage distributionally robust linear programs over Wasserstein balls. Oper. Res. 66(3), 849–869 (2018)

    Article  MathSciNet  Google Scholar 

  20. Hanasusanto, G.A., Kuhn, D., Wiesemann, W.: K-adaptability in two-stage distributionally robust binary programming. Oper. Res. Lett. 44(1), 6–11 (2016)

    Article  MathSciNet  Google Scholar 

  21. Kall, P., Wallace, S.W.: Stochastic Programming, 2nd edn. Wiley, Chichester, UK (1994)

    Google Scholar 

  22. Ling, A., Sun, J., Xiu, N., Yang, X.: Robust two-stage stochastic linear optimization with risk aversion. Eur. J. Oper. Res. 256(1), 215–229 (2017)

    Article  MathSciNet  Google Scholar 

  23. Ling, A., Sun, J., Yang, X.: Robust tracking error portfolio selection with worst-case downside risk measures. J. Econ. Dyn. Control 39, 178–207 (2014)

    Article  MathSciNet  Google Scholar 

  24. Li, B., Qian, X., Sun, J., Teo, K.L., Yu, C.: A model of distributionally robust two-stage stochastic convex programming with linear recourse. Appl. Math. Model. 58, 86–97 (2018)

    Article  MathSciNet  Google Scholar 

  25. Mangasarian, O.L., Shiau, T.H.: A variable-complexity norm maximization problem. SIAM J. Algebraic Discret. Methods 7(3), 455–461 (1986)

    Article  MathSciNet  Google Scholar 

  26. Miller, N., Ruszczyński, A.: Risk-averse two-stage stochastic linear programming: modeling and decomposition. Oper. Res. 59(1), 125–132 (2010)

    Article  MathSciNet  Google Scholar 

  27. Noyan, N.: Risk-averse two-stage stochastic programming with an application to disaster management. Comput. Oper. Res. 39(3), 541–559 (2012)

    Article  MathSciNet  Google Scholar 

  28. Pichler, A., Xu, H.: Quantitative stability analysis for minimax distributionally robust risk optimization. Math. Program. 191, 47–77 (2022)

    Article  MathSciNet  Google Scholar 

  29. Rockafellar, R.T., Uryasev, S.: Optimization of conditional value-at-risk. J. Risk 2, 21–42 (2000)

    Article  Google Scholar 

  30. Scarf, H.E.: A min-max solution of an inventory problem. In: Arrow, K.J., Karlin, S., Scarf, H.E. (eds.) Studies in the Mathematical Theory of Inventory and Production, pp. 201–209. Stanford University Press, Redwood City (1958)

    Google Scholar 

  31. Schultz, R., Tiedemann, S.: Conditional value-at-risk in stochastic programs with mixed-integer recourse. Math. Program. 105(2), 365–386 (2006)

    Article  MathSciNet  Google Scholar 

  32. Shapiro, A.: On duality theory of conic linear problems. In: Goberna, M.Á., López, M.A. (eds.) Semi-Infinite Programming, pp. 135–165. Kluwer Academic Publishers, Norwell (2001)

    Chapter  Google Scholar 

  33. Shapiro, A., Kleywegt, A.: Minimax analysis of stochastic problems. Optim. Methods Softw. 17(3), 523–542 (2002)

    Article  MathSciNet  Google Scholar 

  34. Sion, M.: On general minimax theorems. Pac. J. Math. 8(1), 171–176 (1958)

    Article  MathSciNet  Google Scholar 

  35. Sun, J., Tsai, K.H., Qi, L.: A simplex method for network programs with convex separable piecewise linear costs and its application to stochastic transshipment problems. In: Du, D.Z., Pardalos, P.M. (eds.) Network Optimization Problems: Algorithms, Applications and Complexity, pp. 283–300. World Scientific Publishing Co., London, UK (1993)

    Chapter  Google Scholar 

  36. Wiesemann, W., Kuhn, D., Sim, M.: Distributionally robust convex optimization. Oper. Res. 62(6), 1358–1376 (2014)

    Article  MathSciNet  Google Scholar 

  37. Xie, W.: Tractable reformulations of two-stage distributionally robust linear programs over the type-\(\infty \) Wasserstein ball. Oper. Res. Lett. 48(4), 513–523 (2020)

    Article  MathSciNet  Google Scholar 

  38. Žáčková, J.: On minimax solutions of stochastic linear programming problems. Časopis pro Pěstování Matematiky 91(4), 423–430 (1966)

    Article  MathSciNet  Google Scholar 

  39. Zymler, S., Kuhn, D., Rustem, B.: Distributionally robust joint chance constraints with second-order moment information. Math. Program. 137, 167–198 (2013)

    Article  MathSciNet  Google Scholar 

Download references

Acknowledgements

The authors would like to thank the anonymous reviewers and the handling editors for their constructive comments and valuable suggestions, which helped us to improve the quality of the manuscript.

Author information

Authors and Affiliations

Authors

Corresponding author

Correspondence to Yanjun Wang.

Ethics declarations

Conflict of interest

No potential conflict of interest was reported by the authors.

Additional information

Communicated by Professor Wei Bian.

Publisher's Note

Springer Nature remains neutral with regard to jurisdictional claims in published maps and institutional affiliations.

This research was supported by National Natural Science Foundation of China (Grant No. 11271243).

Appendix: Proofs

Appendix: Proofs

We first draw the following auxiliary result which will be used throughout the proofs of Corollaries \(3.1-3.3\).

Lemma A.1

Suppose that \({\Xi }\subseteq \mathbb {R}^{m}\) is a convex set, then for any \(\varvec{y}_{1},\varvec{y}_{2}\in \mathbb {R}^{m}\) and finite \(\eta \ge 0\), it holds that

$$\begin{aligned} \sup _{\varvec{\xi }\in {\Xi }}\left( \varvec{\xi }^{\top }\varvec{y}_{1}-\eta \left\| \varvec{\xi }-\varvec{y}_{2}\right\| _{p}\right) =\inf _{\left\| \varvec{z}\right\| _{q}\le \eta }\left( \varvec{z}^{\top }\varvec{y}_{2}+ \sup _{\varvec{\xi }\in {\Xi }}\left( (\varvec{y}_{1}-\varvec{z})^{\top } \varvec{\xi }\right) \right) , \end{aligned}$$

where \(\left\| \cdot \right\| _{q}\) is the dual norm of \(\left\| \cdot \right\| _{p}\), i.e., \(\frac{1}{p}+\frac{1}{q}=1\).

Proof

Note that

$$\begin{aligned}&\quad ~\sup _{\varvec{\xi }\in {\Xi }}\left( \varvec{\xi }^{\top }\varvec{y}_{1}-\eta \left\| \varvec{\xi }-\varvec{y}_{2}\right\| _{p}\right) \\&\quad =\sup _{\varvec{\xi }\in {\Xi }}\left( \varvec{\xi }^{\top }\varvec{y}_{1}- \sup _{\left\| \varvec{z}\right\| _{q}\le \eta }{\varvec{z}}^{\top }(\varvec{\xi }-\varvec{y}_{2})\right) \\&\quad =\sup _{\varvec{\xi }\in {\Xi }}\inf _{\left\| \varvec{z}\right\| _{q}\le \eta }\left( \varvec{\xi }^{\top }\varvec{y}_{1}-\varvec{z}^{\top } (\varvec{\xi }-\varvec{y}_{2})\right) \\&\quad =\inf _{\left\| \varvec{z}\right\| _{q}\le \eta }\left( \varvec{z}^{\top }\varvec{y}_{2}+ \sup _{\varvec{\xi }\in {\Xi }}\left( \varvec{\xi }^{\top }\varvec{y}_{1}- \varvec{z}^{\top }\varvec{\xi }\right) \right) , \end{aligned}$$

where the first equality is due to the definition of the dual norm and the third equality follows from the general minimax theorem [34, Corollary 3.3], which can be applied because the set \(\left\{ \varvec{z}\in \mathbb {R}^{m}:{\left\| \varvec{z}\right\| _{q}\le \eta }\right\} \) is compact convex for any finite \(\eta \ge 0\). This completes the proof. \(\square \)

Proof of Corollary 3.1

Note that the support set \({\Xi }=\{\varvec{\xi }\in \mathbb {R}^{m}:({\varvec{\xi }-\varvec{\xi }_{0}})^{\top }\varvec{W}^{-1}({\varvec{\xi }-\varvec{\xi }_{0}})\le 1\}\) with \(\varvec{W}\in \mathbb {S}_{++}^{m}\) is convex. Then by Lemma A.1, we can reformulate (12b) as

$$\begin{aligned} s_{i}&\ge \sup _{\varvec{\xi }\in {\Xi }}\left( \varvec{\xi }^{\top }\varvec{Q} \varvec{\hat{y}}^{i}-\eta \left\| \varvec{\xi }-\varvec{\zeta }^{i}\right\| _{p}\right) +({\varvec{q}^{0}})^{\top }\varvec{\hat{y}}^{i}\nonumber \\&=\inf _{\left\| \varvec{\hat{z}}_{i}\right\| _{q}\le \eta }\left( \varvec{\hat{z}}_{i}^{\top }\varvec{\zeta }^{i}+ \sup _{\varvec{\xi }\in {\Xi }}\left( \varvec{\xi }^{\top }\varvec{Q}\varvec{\hat{y}}^{i}- \varvec{\hat{z}}_{i}^{\top }\varvec{\xi }\right) \right) +(\varvec{q}^{0})^{\top } \varvec{\hat{y}}^{i}. \end{aligned}$$
(37)

We now discuss the reformulation of the inner subproblem \(\sup _{\varvec{\xi }\in {\Xi }}\) in (37). For this problem, there exists \(\varvec{\xi }=\varvec{\xi }_{0}\) such that \(({\varvec{\xi }-\varvec{\xi }_{0}})^{\top }\varvec{W}^{-1}({\varvec{\xi } -\varvec{\xi }_{0}})<1\), which implies Slater’s condition is satisfied, and hence, the strong duality holds. Then we have

$$\begin{aligned} \sup _{\varvec{\xi }\in {\Xi }}\left( \varvec{\xi }^{\top }\varvec{Q}\varvec{\hat{y}}^{i}- \varvec{\hat{z}}_{i}^{\top }\varvec{\xi }\right) =\inf _{\tilde{\nu }_{i}\ge 0}\sup _{\varvec{\xi }}\left( \varvec{\xi }^{\top } \varvec{Q}\varvec{\hat{y}}^{i}-\varvec{\hat{z}}_{i}^{\top }\varvec{\xi }- \hat{\nu }_{i}\left( ({\varvec{\xi }-\varvec{\xi }_{0}})^{\top } \varvec{W}^{-1}({\varvec{\xi }-\varvec{\xi }_{0}})-1\right) \right) , \end{aligned}$$

which can be rewritten as

$$\begin{aligned} \begin{aligned} \min _{\hat{\nu }_{i}\ge 0,\hat{u}_{i}}\quad&\hat{u}_{i}\\ \text{ s.t. }\quad&\varvec{\xi }^{\top }\varvec{Q}\varvec{\hat{y}}^{i}-\varvec{\hat{z}}_{i}^{\top } \varvec{\xi }-\hat{\nu }_{i}\left( ({\varvec{\xi }-\varvec{\xi }_{0}})^{\top } \varvec{W}^{-1}({\varvec{\xi }-\varvec{\xi }_{0}})-1\right) \le \hat{u}_{i}~~\forall \varvec{\xi }\in \mathbb {R}^{m}. \end{aligned} \end{aligned}$$
(38)

Note that the semi-infinite constraint in (38) is equivalent to linear matrix inequality constraint as below:

$$\begin{aligned} \begin{aligned}&\left( \varvec{\xi }^{\top },1\right) \begin{bmatrix} \hat{\nu }_{i}\varvec{W}^{-1}&{}-\frac{1}{2}(2\hat{\nu }_{i}\varvec{W}^{-1} \varvec{\xi }_{0}+\varvec{Q}\varvec{\hat{y}}^{i}-\varvec{\hat{z}}_{i})\\ &{}&{}\\ -\frac{1}{2}(2\hat{\nu }_{i}\varvec{W}^{-1}\varvec{\xi }_{0}+\varvec{Q}\varvec{\hat{y}}^{i}- \varvec{\hat{z}}_{i})^{\top }&{}\hat{u}_{i}+\hat{\nu }_{i}{\varvec{\xi }_{0}}^{\top }\varvec{W}^{-1} \varvec{\xi }_{0}-\hat{\nu }_{i} \end{bmatrix}\\&\quad \left( \varvec{\xi }^{\top },1\right) ^{\top }\ge 0~~\forall \varvec{\xi }\in \mathbb {R}^{m}\\&\iff \begin{bmatrix} \hat{\nu }_{i}\varvec{W}^{-1}&{}-\frac{1}{2}(2\hat{\nu }_{i}\varvec{W}^{-1} \varvec{\xi }_{0}+\varvec{Q}\varvec{\hat{y}}^{i}-\varvec{\hat{z}}_{i})\\ &{}&{}\\ -\frac{1}{2}(2\hat{\nu }_{i}\varvec{W}^{-1}\varvec{\xi }_{0}+\varvec{Q} \varvec{\hat{y}}^{i}-\varvec{\hat{z}}_{i})^{\top }&{}\hat{u}_{i}+\hat{\nu }_{i} {\varvec{\xi }_{0}}^{\top }\varvec{W}^{-1}\varvec{\xi }_{0}-\hat{\nu }_{i} \end{bmatrix}\succeq 0,\nonumber \end{aligned} \end{aligned}$$

which in turn allows us to reformulate problem (38) as

$$\begin{aligned} \begin{aligned} \min _{\hat{\nu }_{i}\ge 0,\hat{u}_{i}}\quad&\hat{u}_{i}\\ \text{ s.t. }\quad&\begin{bmatrix} \hat{\nu }_{i}\varvec{W}^{-1}&{}-\frac{1}{2}(2\hat{\nu }_{i} \varvec{W}^{-1}\varvec{\xi }_{0}+\varvec{Q}\varvec{\hat{y}}^{i} -\varvec{\hat{z}}_{i})\\ &{}&{}\\ -\frac{1}{2}(2\hat{\nu }_{i}\varvec{W}^{-1}\varvec{\xi }_{0}+ \varvec{Q}\varvec{\hat{y}}^{i}-\varvec{\hat{z}}_{i})^{\top }&{}\hat{u}_{i} +\hat{\nu }_{i} {\varvec{\xi }_{0}}^{\top }\varvec{W}^{-1}\varvec{\xi }_{0}-\hat{\nu }_{i} \end{bmatrix}\succeq 0. \end{aligned} \end{aligned}$$
(39)

By replacing the inner subproblem \(\sup _{\varvec{\xi }\in {\Xi }}\) in (37) by (39), we obtain

$$\begin{aligned} \begin{aligned} s_{i}&\ge \sup _{\varvec{\xi }\in {\Xi }}\left( \varvec{\xi }^{\top }\varvec{Q} \varvec{\hat{y}}^{i}-\eta \left\| \varvec{\xi }-\varvec{\zeta }^{i}\right\| _{p}\right) +({\varvec{q}^{0}})^{\top }\varvec{\hat{y}}^{i}\\&=\inf _{\varvec{\hat{z}}_{i},\hat{\nu }_{i},\hat{u}_{i}} \left( \varvec{\hat{z}}_{i}^{\top } \varvec{\zeta }^{i}+\hat{u}_{i}\right) +(\varvec{q}^{0})^{\top }\varvec{\hat{y}}^{i}\\&\qquad \text{ s.t. } \begin{bmatrix} \hat{\nu }_{i}\varvec{W}^{-1}&{}-\frac{1}{2}(2\hat{\nu }_{i} \varvec{W}^{-1}\varvec{\xi }_{0}+\varvec{Q}\varvec{\hat{y}}^{i}-\varvec{\hat{z}}_{i})\\ &{}&{}\\ -\frac{1}{2}(2\hat{\nu }_{i}\varvec{W}^{-1}\varvec{\xi }_{0}+ \varvec{Q}\varvec{\hat{y}}^{i}-\varvec{\hat{z}}_{i})^{\top }&{}\hat{u}_{i}+ \hat{\nu }_{i}{\varvec{\xi }_{0}}^{\top }\varvec{W}^{-1}\varvec{\xi }_{0}-\hat{\nu }_{i}\\ \end{bmatrix}\succeq 0\\ \nonumber&\qquad \quad ~\;\left\| \varvec{\hat{z}}_{i}\right\| _{q}\le \eta \\&\qquad \quad ~\;\hat{\nu }_{i}\ge 0. \end{aligned} \end{aligned}$$

Similarly, for the reformulation of (12c), we present the following parallel conclusion.

$$\begin{aligned} \begin{aligned} s_{i}+\frac{\lambda \beta }{\alpha }&\ge \sup _{\varvec{\xi }\in {\Xi }} \left( (1+\frac{\lambda }{\alpha })\varvec{\xi }^{\top }\varvec{Q}\varvec{\tilde{y}}^{i}-\eta \left\| \varvec{\xi }-\varvec{\zeta }^{i}\right\| _{p}\right) +(1+\frac{\lambda }{\alpha })({\varvec{q}^{0}})^{\top }\varvec{\tilde{y}}^{i}\\&=\inf _{\varvec{\tilde{z}}_{i},\tilde{\nu }_{i},\tilde{u}_{i}} \left( \varvec{\tilde{z}}_{i}^{\top }\varvec{\zeta }^{i}+\tilde{u}_{i}\right) + (1+\frac{\lambda }{\alpha })({\varvec{q}^{0}})^{\top }\varvec{\tilde{y}}^{i}\\&\qquad \text{ s.t. } \begin{bmatrix} \tilde{\nu }_{i}\varvec{W}^{-1}&{}\!\!\!\!\!\!\!\!\!\!\!\!\!\!\!-\frac{1}{2} (2\tilde{\nu }_{i}\varvec{W}^{-1}\varvec{\xi }_{0}+(1+\frac{\lambda }{\alpha }) \varvec{Q}\varvec{\tilde{y}}^{i}-\varvec{\tilde{z}}_{i})\\ &{}&{}\\ -\frac{1}{2}(2\tilde{\nu }_{i}\varvec{W}^{-1}\varvec{\xi }_{0}+ (1+\frac{\lambda }{\alpha })\varvec{Q}\varvec{\tilde{y}}^{i}-\varvec{\tilde{z}}_{i})^{\top }&{} \tilde{u}_{i}+\tilde{\nu }_{i}{\varvec{\xi }_{0}}^{\top }\varvec{W}^{-1}\varvec{\xi }_{0}-\tilde{\nu }_{i}\\ \end{bmatrix}\succeq 0\\&\qquad \quad ~\;\left\| \varvec{\tilde{z}}_{i}\right\| _{q}\le \eta \\&\qquad \quad ~\;\tilde{\nu }_{i}\ge 0. \end{aligned} \end{aligned}$$

Hence, the claim follows. \(\square \)

Proof of Corollary 3.2

Note that the support set \({\Xi }=\{\varvec{\xi }\in \mathbb {R}^{m}:\varvec{C}\varvec{\xi }\le \varvec{d}\}\) with \(\varvec{C}\in \mathbb {R}^{k\times m}\) and \(\varvec{d}\in \mathbb {R}^{k}\) is convex. Then by Lemma A.1, we can reformulate (12b) as

$$\begin{aligned} s_{i}&\ge \sup _{\varvec{\xi }\in {\Xi }}\left( \varvec{\xi }^{\top } \varvec{Q}\varvec{\hat{y}}^{i}-\eta \left\| \varvec{\xi }-\varvec{\zeta }^{i}\right\| _{p}\right) +({\varvec{q}^{0}})^{\top }\varvec{\hat{y}}^{i}\nonumber \\&=\inf _{\left\| \varvec{\hat{z}}_{i}\right\| _{q}\le \eta }\left( \varvec{\hat{z}}_{i}^{\top }\varvec{\zeta }^{i}+ \sup _{\varvec{\xi }\in {\Xi }}\left( \varvec{\xi }^{\top }\varvec{Q}\varvec{\hat{y}}^{i} -\varvec{\hat{z}}_{i}^{\top }\varvec{\xi }\right) \right) +(\varvec{q}^{0})^{\top } \varvec{\hat{y}}^{i}. \end{aligned}$$
(40)

We now discuss the reformulation of the inner linear programming subproblem \(\sup _{\varvec{\xi }\in {\Xi }}\) in (40). For this problem, the strong duality theory yields

$$\begin{aligned} \sup _{\varvec{\xi }\in {\Xi }}\left( \varvec{\xi }^{\top }\varvec{Q}\varvec{\hat{y}}^{i}- \varvec{\hat{z}}_{i}^{\top }\varvec{\xi }\right) = \left\{ \begin{aligned} \sup _{\varvec{\xi }}\quad&(\varvec{Q}\varvec{\hat{y}}^{i}-\varvec{\hat{z}}_{i})^{\top }\varvec{\xi }\\ \text{ s.t. }\quad&\varvec{C}\varvec{\xi }\le \varvec{d} \end{aligned} \right. = \left\{ \begin{aligned} \inf _{\varvec{\gamma }_{i}\ge 0}\quad&\varvec{\gamma }_{i}^{\top }\varvec{d}\\ \text{ s.t. }\quad&\varvec{C}^{\top }\varvec{\gamma }_{i}=\varvec{Q}\varvec{\hat{y}}^{i}-\varvec{\hat{z}}_{i}. \end{aligned} \right. \end{aligned}$$
(41)

By replacing the inner subproblem \(\sup _{\varvec{\xi }\in {\Xi }}\) in (40) by (41), we have

$$\begin{aligned} \begin{aligned} s_{i}&\ge \sup _{\varvec{\xi }\in {\Xi }}\left( \varvec{\xi }^{\top } \varvec{Q}\varvec{\hat{y}}^{i}-\eta \left\| \varvec{\xi }-\varvec{\zeta }^{i}\right\| _{p}\right) +({\varvec{q}^{0}})^{\top }\varvec{\hat{y}}^{i}\\ \nonumber&=\inf _{\varvec{\hat{z}}_{i},\varvec{\gamma }_{i}} \left( \varvec{\hat{z}}_{i}^{\top }\varvec{\zeta }^{i}+\varvec{\gamma }_{i}^{\top } \varvec{d}\right) +({\varvec{q}^{0}})^{\top }\varvec{\hat{y}}^{i}\\ \nonumber&~~~~~\text{ s.t. }~~\;\varvec{\hat{z}}_{i}=\varvec{Q}\varvec{\hat{y}}^{i}- \varvec{C}^{\top }\varvec{\gamma }_{i}\\ \nonumber&\qquad \quad ~\;\left\| \varvec{\hat{z}}_{i}\right\| _{q}\le \eta \\ \nonumber&\qquad \quad ~\;\;\varvec{\gamma }_{i}\ge 0.\nonumber \end{aligned} \end{aligned}$$

Similarly, for the reformulation of (12c), we present the following parallel conclusion.

$$\begin{aligned} \begin{aligned} s_{i}+\frac{\lambda \beta }{\alpha }&\ge \sup _{\varvec{\xi }\in {\Xi }} \left( (1+\frac{\lambda }{\alpha })\varvec{\xi }^{\top }\varvec{Q}\varvec{\tilde{y}}_{i}-\eta \left\| \varvec{\xi }-\varvec{\zeta }^{i}\right\| _{p}\right) +(1+\frac{\lambda }{\alpha })({\varvec{q}^{0}})^{\top } \varvec{\tilde{y}}_{i}\\ \nonumber&=\inf _{\varvec{\tilde{z}}_{i},\varvec{\mu }_{i}} \left( \varvec{\tilde{z}}_{i}^{\top }\varvec{\zeta }^{i}+\varvec{\mu }_{i}^{\top } \varvec{d}\right) +(1+\frac{\lambda }{\alpha })({\varvec{q}^{0}})^{\top }\varvec{\tilde{y}}_{i}\\ \nonumber&~~~~~\text{ s.t. }~~\;\varvec{\tilde{z}}_{i}=(1+\frac{\lambda }{\alpha }) \varvec{Q}\varvec{\tilde{y}}_{i}-\varvec{C}^{\top }\varvec{\mu }_{i}\\ \nonumber&\qquad \quad ~\;\left\| \varvec{\tilde{z}}_{i}\right\| _{q}\le \eta \\ \nonumber&\qquad \quad ~\;\;\varvec{\mu }_{i}\ge 0.\nonumber \end{aligned} \end{aligned}$$

Hence, the claim follows. \(\square \)

Proof of Corollary 3.3

Note that the support set \({\Xi }=\mathbb {R}^{m}\) is convex. Then by Lemma A.1, we can reformulate (12b) as

$$\begin{aligned} s_{i}&\ge \sup _{\varvec{\xi }\in {\Xi }}\left( \varvec{\xi }^{\top }\varvec{Q} \varvec{\hat{y}}^{i}-\eta \left\| \varvec{\xi }-\varvec{\zeta }^{i}\right\| _{p}\right) +({\varvec{q}^{0}})^{\top }\varvec{\hat{y}}^{i}\nonumber \\&=\inf _{\left\| \varvec{\hat{z}}_{i}\right\| _{q}\le \eta }\left( \varvec{\hat{z}}_{i}^{\top }\varvec{\zeta }^{i}+ \sup _{\varvec{\xi }\in {\Xi }}\left( \varvec{\xi }^{\top }\varvec{Q}\varvec{\hat{y}}^{i}- \varvec{\hat{z}}_{i}^{\top }\varvec{\xi }\right) \right) +(\varvec{q}^{0})^{\top } \varvec{\hat{y}}^{i}. \end{aligned}$$
(42)

We now discuss the reformulation of the inner unconstrained subproblem \(\sup _{\varvec{\xi }\in {\Xi }}\) in (42), which implies that \(\varvec{\hat{z}}_{i}=\varvec{Q}\varvec{\hat{y}}^{i}\), and then (12b) is equivalent to

$$\begin{aligned} \begin{aligned}&(\varvec{Q}\varvec{\hat{y}}^{i})^{\top }\varvec{\zeta }^{i}+ ({\varvec{q}^{0}})^{\top }\varvec{\hat{y}}^{i}\le s_{i}\\&\left\| \varvec{Q}\varvec{\hat{y}}^{i} \right\| _{q}\le \eta . \end{aligned} \end{aligned}$$

Similarly, for the reformulation of (12c), we present the following parallel conclusion.

$$\begin{aligned} \begin{aligned}&(1+\frac{\lambda }{\alpha })\left( (\varvec{Q}\varvec{\tilde{y}}^{i})^{\top } \varvec{\zeta }^{i}+({\varvec{q}^{0}})^{\top }\varvec{\tilde{y}}^{i}\right) \le s_{i}+\frac{\lambda \beta }{\alpha }\\&(1+\frac{\lambda }{\alpha })\left\| \varvec{Q}\varvec{\tilde{y}}^{i}\right\| _{q}\le \eta . \end{aligned} \end{aligned}$$

Hence, the claim follows. \(\square \)

Proof of Proposition 5.1

Note that by using (6), the worst-case mean-CVaR problem in (28) can be rewritten as

$$\begin{aligned}&\sup _{\mathbb {P}\in \mathcal {P}}\left\{ \mathbb {E}_{\mathbb {P}} \left[ Q(\varvec{x},\varvec{\tilde{\xi }})\right] +\lambda \textrm{CVaR}_{1-\alpha } (Q(\varvec{x},\varvec{\tilde{\xi }}))\right\}&\nonumber \\&\quad =\sup _{\mathbb {P}\in \mathcal {P}}\left\{ \mathbb {E}_{\mathbb {P}} \left[ Q(\varvec{x},\varvec{\tilde{\xi }})\right] +\lambda \inf _{\beta \in \mathbb {R}}\left\{ \beta +\frac{1}{\alpha } \mathbb {E}_{\mathbb {P}}\left[ \left( Q\left( \varvec{x}, \varvec{\tilde{\xi }}\right) -\beta \right) _{+}\right] \right\} \right\} \end{aligned}$$
(43a)
$$\begin{aligned}&\quad =\sup _{\mathbb {P}\in \mathcal {P}}\inf _{\beta \in \mathbb {R}}\left\{ \lambda \beta +\mathbb {E}_{\mathbb {P}} \left[ Q(\varvec{x},\varvec{\tilde{\xi }})+\frac{\lambda }{\alpha } \left( Q\left( \varvec{x},\varvec{\tilde{\xi }}\right) -\beta \right) _{+}\right] \right\} \end{aligned}$$
(43b)
$$\begin{aligned}&\quad =\inf _{\beta \in \mathbb {R}}\left\{ \lambda \beta + \sup _{\mathbb {P}\in \mathcal {P}}\mathbb {E}_{\mathbb {P}} \left[ Q(\varvec{x},\varvec{\tilde{\xi }})+\frac{\lambda }{\alpha } \left( Q\left( \varvec{x},\varvec{\tilde{\xi }}\right) -\beta \right) _{+}\right] \right\} , \end{aligned}$$
(43c)

where the third equality is due to a stochastic saddle point theorem [33].

The subordinate worst-case expectation problem in (43c) can be expressed as

$$\begin{aligned} \sup _{\mathbb {P}\in \mathcal {M}_{+}}~&\int _{\widehat{{\Xi }}}Q(\varvec{x},\varvec{\xi }) +\frac{\lambda }{\alpha }\left( Q\left( \varvec{x},\varvec{\xi }\right) -\beta \right) _{+}\mathbb {P} \left( d \varvec{\xi }\right) \end{aligned}$$
(44a)
$$\begin{aligned} \text{ s.t. }\quad \!\!&\int _{\widehat{{\Xi }}}\mathbb {P}\left( d \varvec{\xi }\right) =1 \end{aligned}$$
(44b)
$$\begin{aligned}&\int _{\widehat{{\Xi }}}\varvec{\xi }\mathbb {P}\left( d \varvec{\xi }\right) =\varvec{\hat{\mu }} \end{aligned}$$
(44c)
$$\begin{aligned}&\int _{\widehat{{\Xi }}}\varvec{\xi }\varvec{\xi }^{\top }\mathbb {P}\left( d \varvec{\xi }\right) = \varvec{\widehat{{\Sigma }}}+\varvec{\hat{\mu }}\varvec{\hat{\mu }}^{\top }, \end{aligned}$$
(44d)

where \(\mathcal {M}_{+}\) represents the one of nonnegative Borel measures on \(\mathbb {R}^{mn}\).

We now assign dual variables \(z_{0}\in \mathbb {R}\), \(\varvec{z}\in \mathbb {R}^{mn}\), and \(\varvec{Z}\in \mathbb {S}^{mn}_{+}\) with the constraints of the primal problem (44), and then obtain the following dual problem

$$\begin{aligned} \inf _{z_{0},\varvec{z},\varvec{Z}} \quad&z_{0}+\varvec{z}^{\top }\varvec{\hat{\mu }}+ \langle \varvec{Z},\varvec{\widehat{{\Sigma }}}+\varvec{\hat{\mu }} \varvec{\hat{\mu }}^{\top }\rangle \end{aligned}$$
(45a)
$$\begin{aligned} \text{ s.t. }\quad&z_{0}+\varvec{z}^{\top }\varvec{\xi }+ \langle \varvec{Z},\varvec{\xi }\varvec{\xi }^{\top }\rangle \ge Q(\varvec{x},\varvec{\xi })+\frac{\lambda }{\alpha } \left( Q\left( \varvec{x},\varvec{\xi }\right) - \beta \right) _{+}~~\forall \varvec{\xi }\in \widehat{{\Xi }} \end{aligned}$$
(45b)
$$\begin{aligned}&z_{0}\in \mathbb {R},~\varvec{z}\in \mathbb {R}^{mn},~\varvec{Z}\in \mathbb {S}^{mn}_{+}. \end{aligned}$$
(45c)

It can be shown that strong duality holds due to \(\varvec{\widehat{{\Sigma }}}\succ 0\) [32]. Note that constraint (45b) can be rewritten as

$$\begin{aligned} \sup _{\varvec{\xi }\in \widehat{{\Xi }}} \left( Q(\varvec{x},\varvec{\xi })+\frac{\lambda }{\alpha } \left( Q\left( \varvec{x},\varvec{\xi }\right) - \beta \right) _{+}-z_{0}-\varvec{z}^{\top }\varvec{\xi }- \langle \varvec{Z},\varvec{\xi }\varvec{\xi }^{\top }\rangle \right) \le 0, \end{aligned}$$

then by (29), which is further equivalent to

$$\begin{aligned} \begin{aligned}&\sup _{\varvec{\xi }\in \widehat{{\Xi }}} \left( \varvec{\xi }^{\top }\varvec{y}^{1} -z_{0}-\varvec{z}^{\top }\varvec{\xi }- \langle \varvec{Z},\varvec{\xi }\varvec{\xi }^{\top }\rangle \right) \le 0\\&\sup _{\varvec{\xi }\in \widehat{{\Xi }}} \left( \varvec{\xi }^{\top }\varvec{y}^{2}+\frac{\lambda }{\alpha } \left( \varvec{\xi }^{\top }\varvec{y}^{2}- \beta \right) -z_{0}-\varvec{z}^{\top }\varvec{\xi }- \langle \varvec{Z},\varvec{\xi }\varvec{\xi }^{\top }\rangle \right) \le 0. \end{aligned} \end{aligned}$$
(46)

For the optimization problem \(\sup _{\varvec{\xi }\in \widehat{{\Xi }}}\) on the left-hand side of the first inequality in the constraint system (46), there exists \(\varvec{\xi }=\varvec{\xi }_{0}\) such that \(({\varvec{\xi }-\varvec{\xi }_{0}})^{\top }\varvec{W}^{-1}({\varvec{\xi }-\varvec{\xi }_{0}})<1\), which implies Slater’s condition is satisfied, and hence, the strong duality holds. Then we have

$$\begin{aligned}&\sup _{\varvec{\xi }\in \widehat{{\Xi }}} \left( \varvec{\xi }^{\top }\varvec{y}^{1} -z_{0}-\varvec{z}^{\top }\varvec{\xi }- \langle \varvec{Z},\varvec{\xi }\varvec{\xi }^{\top }\rangle \right) \\&\quad =\inf _{\nu _{1}\ge 0}\sup _{\varvec{\xi }}\left( \varvec{\xi }^{\top }\varvec{y}^{1} -z_{0}-\varvec{z}^{\top }\varvec{\xi }- \langle \varvec{Z},\varvec{\xi }\varvec{\xi }^{\top }\rangle -\nu _{1} \left( ({\varvec{\xi }-\varvec{\xi }_{0}})^{\top }\varvec{W}^{-1}({\varvec{\xi }- \varvec{\xi }_{0}})-1 \right) \right) ,\nonumber \end{aligned}$$

which can be converted into

$$\begin{aligned} \begin{aligned} \min _{\nu _{1}\ge 0,u_{1}}\quad&u_{1}\\ \text{ s.t. }\quad&\varvec{\xi }^{\top }\varvec{y}^{1} -z_{0}-\varvec{z}^{\top }\varvec{\xi }- \langle \varvec{Z},\varvec{\xi }\varvec{\xi }^{\top }\rangle -\nu _{1} \left( ({\varvec{\xi }-\varvec{\xi }_{0}})^{\top }\varvec{W}^{-1} ({\varvec{\xi }-\varvec{\xi }_{0}})-1 \right) \le u_{1}~~\forall \varvec{\xi }\in \mathbb {R}^{mn}. \end{aligned} \end{aligned}$$
(47)

Note that the semi-infinite constraint in (47) is equivalent to linear matrix inequality constraint as below:

$$\begin{aligned} \begin{aligned}&\left( \varvec{\xi }^{\top },1\right) \begin{bmatrix} \varvec{Z}+\nu _{1}\varvec{W}^{-1}&{}-\frac{1}{2}(2\nu _{1}\varvec{W}^{-1} \varvec{\xi }_{0}+\varvec{y}^{1}-\varvec{z})\\ &{}&{}\\ -\frac{1}{2}(2\nu _{1}\varvec{W}^{-1}\varvec{\xi }_{0}+ \varvec{y}^{1}-\varvec{z})^{\top }&{}~~~u_{1}+\nu _{1}{\varvec{\xi }_{0}}^{\top } \varvec{W}^{-1}\varvec{\xi }_{0}+z_{0}-\nu _{1} \end{bmatrix}\\&\quad \left( \varvec{\xi }^{\top },1\right) ^{\top }\ge 0~~\forall \varvec{\xi }\in \mathbb {R}^{mn}\\&\iff \begin{bmatrix} \varvec{Z}+\nu _{1}\varvec{W}^{-1}&{}-\frac{1}{2}(2\nu _{1}\varvec{W}^{-1}\varvec{\xi }_{0}+ \varvec{y}^{1}-\varvec{z})^{\top }\\ &{}&{}\\ -\frac{1}{2}(2\nu _{1}\varvec{W}^{-1}\varvec{\xi }_{0}+\varvec{y}^{1}- \varvec{z})^{\top }&{}~~~u_{1}+\nu _{1}{\varvec{\xi }_{0}}^{\top }\varvec{W}^{-1}\varvec{\xi }_{0}+ z_{0}-\nu _{1} \end{bmatrix}\succeq 0,\nonumber \end{aligned} \end{aligned}$$

which in turn allows us to reformulate problem (47) as

$$\begin{aligned} \begin{aligned} \min _{\nu _{1}\ge 0,u_{1}}\quad&u_{1}\\ \nonumber \text{ s.t. }\quad&\begin{bmatrix} \varvec{Z}+\nu _{1}\varvec{W}^{-1}&{}-\frac{1}{2}(2\nu _{1}\varvec{W}^{-1} \varvec{\xi }_{0}+\varvec{y}^{1}-\varvec{z})\\ &{}&{}\\ -\frac{1}{2}(2\nu _{1}\varvec{W}^{-1} \varvec{\xi }_{0}+\varvec{y}^{1}-\varvec{z})^{\top }&{}~~~u_{1}+ \nu _{1}{\varvec{\xi }_{0}}^{\top }\varvec{W}^{-1}\varvec{\xi }_{0}+z_{0}-\nu _{1} \end{bmatrix}\succeq 0. \end{aligned} \end{aligned}$$

Therefore, we obtain

$$\begin{aligned} \begin{aligned} 0&\ge \sup _{\varvec{\xi }\in \widehat{{\Xi }}} \left( \varvec{\xi }^{\top }\varvec{y}^{1} -z_{0}-\varvec{z}^{\top }\varvec{\xi }- \langle \varvec{Z},\varvec{\xi }\varvec{\xi }^{\top }\rangle \right) \\ \nonumber&=\min _{\nu _{1},u_{1}}\quad u_{1}\\ \nonumber&\qquad \text{ s.t. } \begin{bmatrix} \varvec{Z}+\nu _{1}\varvec{W}^{-1}&{}-\frac{1}{2}(2\nu _{1} \varvec{W}^{-1}\varvec{\xi }_{0}+\varvec{y}^{1}-\varvec{z})\\ &{}&{}\\ -\frac{1}{2}(2\nu _{1}\varvec{W}^{-1}\varvec{\xi }_{0}+ \varvec{y}^{1}-\varvec{z})^{\top }&{}~~~u_{1}+\nu _{1}{\varvec{\xi }_{0}}^{\top } \varvec{W}^{-1}\varvec{\xi }_{0}+z_{0}-\nu _{1} \end{bmatrix}\succeq 0\\ \nonumber&\qquad \quad ~\;\nu _{1}\ge 0.\nonumber \end{aligned} \end{aligned}$$

Similarly, for the reformulation of the second inequality in the constraint system (46), we present the following parallel conclusion.

$$\begin{aligned} \begin{aligned} \frac{\lambda \beta }{\alpha }&\ge \sup _{\varvec{\xi }\in \widehat{{\Xi }}} \left( (1+\frac{\lambda }{\alpha })\varvec{\xi }^{\top }\varvec{y}^{2} -z_{0}-\varvec{z}^{\top }\varvec{\xi }- \langle \varvec{Z},\varvec{\xi }\varvec{\xi }^{\top }\rangle \right) \\ \nonumber&=\min _{\nu _{2},u_{2}}\quad u_{2}\\ \nonumber&\qquad \text{ s.t. } \begin{bmatrix} \varvec{Z}+\nu _{2}\varvec{W}^{-1}&{}-\frac{1}{2}(2\nu _{2} \varvec{W}^{-1}\varvec{\xi }_{0}+(1+\frac{\lambda }{\alpha })\varvec{y}^{2}- \varvec{z})\\ &{}&{}\\ -\frac{1}{2}(2\nu _{2}\varvec{W}^{-1}\varvec{\xi }_{0}+(1+\frac{\lambda }{\alpha }) \varvec{y}^{2}-\varvec{z})^{\top }&{}~~~u_{2}+\nu _{2} {\varvec{\xi }_{0}}^{\top }\varvec{W}^{-1}\varvec{\xi }_{0}+z_{0}-\nu _{2} \end{bmatrix}\succeq 0\\ \nonumber&\qquad \quad ~\;\nu _{2}\ge 0.\nonumber \end{aligned} \end{aligned}$$

In conclusion, problem (28) with moment-based ambiguity set (31) is equivalent to

$$\begin{aligned} \min _{\varvec{x},\varvec{y}^{1},\varvec{y}^{2},\varvec{z},\varvec{Z},z_{0},\beta ,\nu _{1},\nu _{2},u_{1},u_{2}} \quad&\left\{ \varvec{c}^{\top }\varvec{x}+\lambda \beta +z_{0}+\varvec{z}^{\top }\varvec{\hat{\mu }}+ \langle \varvec{Z},\varvec{\widehat{{\Sigma }}}+ \varvec{\hat{\mu }}\varvec{\hat{\mu }}^{\top }\rangle \right\} \\&\text{ s.t. }~~~~~~\quad \begin{bmatrix} \varvec{Z}+\nu _{1}\varvec{W}^{-1}&{}-\frac{1}{2}(2\nu _{1} \varvec{W}^{-1}\varvec{\xi }_{0}+\varvec{y}^{1}-\varvec{z})\\ &{}&{}\\ -\frac{1}{2}(2\nu _{1}\varvec{W}^{-1} \varvec{\xi }_{0}+\varvec{y}^{1}-\varvec{z})^{\top }&{}~~~u_{1}+\nu _{1}{\varvec{\xi }_{0}}^{\top } \varvec{W}^{-1}\varvec{\xi }_{0}+z_{0}-\nu _{1} \end{bmatrix}\succeq 0 \\&\begin{bmatrix} \varvec{Z}+\nu _{2}\varvec{W}^{-1}&{}-\frac{1}{2}(2\nu _{2}\varvec{W}^{-1} \varvec{\xi }_{0}+(1+\frac{\lambda }{\alpha })\varvec{y}^{2}- \varvec{z})\\ &{}&{}\\ -\frac{1}{2}(2\nu _{2}\varvec{W}^{-1}\varvec{\xi }_{0}+ (1+\frac{\lambda }{\alpha })\varvec{y}^{2}-\varvec{z})^{\top }&{}~~~u_{2}+\nu _{2} {\varvec{\xi }_{0}}^{\top }\varvec{W}^{-1}\varvec{\xi }_{0}+z_{0}-\nu _{2} \end{bmatrix}\succeq 0 \\&\sum _{j=1}^{n}y^{1}_{rj}=x_{r},~\sum _{j=1}^{n}y^{2}_{rj}=x_{r}~~\forall r\in [m]\\&\sum _{r=1}^{m}y^{1}_{rj}=a_{j},~\sum _{r=1}^{m}y^{2}_{rj}=a_{j}~~\forall j\in [n]\\&u_{1}\le 0,u_{2}\le \frac{\lambda \beta }{\alpha }\\&\varvec{x}\ge 0,~\varvec{y}^{1}\ge 0,~\varvec{y}^{2}\ge 0, ~\nu _{1}\ge 0,~\nu _{2}\ge 0,~\varvec{Z}\in \mathbb {S}^{mn}_{+}. \end{aligned}$$

This completes the proof. \(\square \)

Rights and permissions

Springer Nature or its licensor (e.g. a society or other partner) holds exclusive rights to this article under a publishing agreement with the author(s) or other rightsholder(s); author self-archiving of the accepted manuscript version of this article is solely governed by the terms of such publishing agreement and applicable law.

Reprints and permissions

About this article

Check for updates. Verify currency and authenticity via CrossMark

Cite this article

Gu, Y., Huang, Y. & Wang, Y. Data-Driven Distributionally Robust Risk-Averse Two-Stage Stochastic Linear Programming over Wasserstein Ball. J Optim Theory Appl 200, 242–279 (2024). https://doi.org/10.1007/s10957-023-02331-z

Download citation

  • Received:

  • Accepted:

  • Published:

  • Issue Date:

  • DOI: https://doi.org/10.1007/s10957-023-02331-z

Keywords

Mathematics Subject Classification

Navigation